4
$\begingroup$

There are quite a few simple results about convergent/divergent series derived from similar ones. Here is a question in the same spirit that I saw posted on another forum. Unfortunately, I don't have any background information on the problem:

Given a non-negative sequence $\{a_n\}_{n=1}^\infty$ such that $\sum_{n=1}^{\infty} a_n^2<\infty$, is it true that the derived series $$\sum_{n=1}^{\infty}\left(\sum_{k=1}^{\infty}\frac{a_{kn}}{k} \right)^2$$ must also converge?

At first look, the statement looks false (to me), but I have not been able to find a counter example. Maybe it's trivial but it doesn't look intuitive to me, so I would appreciate any suggestions.

$\endgroup$
5
  • $\begingroup$ How is $a_{kn}$ related to $a_n$ ? $\endgroup$ Commented Nov 29, 2023 at 23:13
  • 4
    $\begingroup$ @ChristianRemling Ha! That's why I hate when people write $a_{ij}$ instead of $a_{i,j}$ for the entries of a matrix, for instance. In this case, the OP did not abuse notation. The number $a_{kn}$ is the $(kn)^{\text{th}}$ element of the sequence $(a_n)_n$. $\endgroup$ Commented Nov 29, 2023 at 23:24
  • $\begingroup$ @Christian Remling: presumably $kn$ in $a_{kn}$ is the product of $k$ and $n$. $\endgroup$ Commented Nov 29, 2023 at 23:25
  • $\begingroup$ Correct, $a_{kn}$ is the $(kn)^{th}$ term of the series. $\endgroup$ Commented Nov 29, 2023 at 23:53
  • 3
    $\begingroup$ If the sequence is submultiplicative, $a_{kn}\le a_ka_n$, then by Cauchy-Schwarz the derived series is bounded in terms of the first one (maybe this gives a hint for a counterexample) $\endgroup$ Commented Nov 30, 2023 at 0:00

1 Answer 1

6
$\begingroup$

I think the answer is no.

By functional analysis, we know that it suffices to show the existence of a sequence $a_n$ such that $\lVert a \rVert_2 = 1$ and $$\sum_{n = 1}^{\infty} \left(\sum_{j = 1}^\infty \frac{a_{jn}}{j}\right)^2$$ is arbitrarily large.

Take the first $k$ prime numbers $p_1, \cdots, p_k$, and an integer $s\ge1$. Let $\mathbb{B}_{k, s}$ denote the set $$\mathbb{B}_{k, s} = \{p_1^{i_1} p_2^{i_2} \cdots p_k^{i_k}: 0 \leq i_1, i_2, \cdots, i_k \leq s\}.$$ We consider the sequence $$a_n = \begin{cases} \frac{1}{\sqrt{|\mathbb{B}_{k, s}|}}\text{ if } n \in \mathbb{B}_{k, s}, \\ 0 \text{ otherwise}. \end{cases}.$$ Then $\lVert{a\rVert}_2 = 1$. However, if $n \in \mathbb{B}_{k, s - 1}$, then $np_i \in \mathbb{B}_{k, s}$ for each $i \in [k]$. So we have $$\sum_{j = 1}^\infty \frac{a_{jn}}{j} \geq \left(\sum_{i = 1}^k \frac{1}{p_i}\right) \cdot \frac{1}{\sqrt{|\mathbb{B}_{k, s}|}}.$$ Thus $$\sum_{n = 1}^{\infty} \left(\sum_{j = 1}^\infty \frac{a_{jn}}{j}\right)^2 \geq \left(\sum_{i = 1}^k \frac{1}{p_i}\right)^2 \frac{|\mathbb{B}_{k, s - 1}|}{|\mathbb{B}_{k, s}|} = \left(\sum_{i = 1}^k \frac{1}{p_i}\right)^2 \cdot \frac{s^k}{(s+1)^k} \gg (\log\log k)^2 \cdot \frac{s^k}{(s+1)^k}.$$ which can be arbitrarily large by tuning $s$ and $k$, as desired.

$\endgroup$

You must log in to answer this question.

Start asking to get answers

Find the answer to your question by asking.

Ask question

Explore related questions

See similar questions with these tags.